Bài tập  /  Bài đang cần trả lời

Từ mặt nước trong một bể nước, tại ba vị trí đôi một cách nhau 2 m, người ta lần lượt thả dây dọi để quả dọi chạm đáy bể. Phần dây dọi (thẳng) nằm trong nước tại ba vị trí đó lần lượt có độ dài 4 m; 4,4 m; 4,8 m. Biết đáy bể là phẳng. Hỏi đáy bể nghiêng so với mặt phẳng nằm ngang một góc bao nhiêu độ?

Từ mặt nước trong một bể nước, tại ba vị trí đôi một cách nhau 2 m, người ta lần lượt thả dây dọi để quả dọi chạm đáy bể. Phần dây dọi (thẳng) nằm trong nước tại ba vị trí đó lần lượt có độ dài 4 m; 4,4 m; 4,8 m. Biết đáy bể là phẳng. Hỏi đáy bể nghiêng so với mặt phẳng nằm ngang một góc bao nhiêu độ?

1 trả lời
Hỏi chi tiết
1
0
0
CenaZero♡
11/09 11:10:55

Gọi 3 điểm ở trên mặt nước lần lượt là A, B, C và ba điểm tương ứng ở đáy bể là A', B', C' sao cho AA' = 4 m, BB' = 4,4 m, CC' = 4,8 m.

Chọn hệ trục tọa độ như hình vẽ, O là trung điểm của AC.

Ta có A(0; 1; 0), \(B\left( {\sqrt 3 ;0;0} \right)\), C(0; −1; 0), \(A'(0;1;4)\), \(B'\left( {\sqrt 3 ;0;4,4} \right)\), C'(0; −1; 4,8).

Ta có \(\overrightarrow {A'B'} = \left( {\sqrt 3 ; - 1;0,4} \right)\), \(\overrightarrow {A'C'} = \left( {0; - 2;0,8} \right)\).

Có \(\left[ {\overrightarrow {A'B'} ,\overrightarrow {A'C'} } \right] = \left( {\left| {\begin{array}{*{20}{c}}{ - 1}&{0,4}\\{ - 2}&{0,8}\end{array}} \right|,\left| {\begin{array}{*{20}{c}}{0,4}&{\sqrt 3 }\\{0,8}&0\end{array}} \right|,\left| {\begin{array}{*{20}{c}}{\sqrt 3 }&{ - 1}\\0&{ - 2}\end{array}} \right|} \right)\) \( = \left( {0; - 0,8\sqrt 3 ; - 2\sqrt 3 } \right)\).

Mặt phẳng đáy bể là mặt phẳng (A'B'C') có một vectơ pháp tuyến là \(\overrightarrow n = \left( {0; - 0,8\sqrt 3 ; - 2\sqrt 3 } \right)\).

Mặt phẳng nằm ngang (mặt nước) chính là mặt phẳng Oxy: z = 0 có một vectơ pháp tuyến là \(\overrightarrow k = \left( {0;0;1} \right)\).

Do đó \(\cos \left( {\left( {A'B'C'} \right),\left( {Oxy} \right)} \right) = \frac{{\left| {0.0 - 0,8\sqrt 3 .0 - 2\sqrt 3 .1} \right|}}{{\sqrt {{0^2} + {{\left( { - 0,8\sqrt 3 } \right)}^2} + {{\left( { - 2\sqrt 3 } \right)}^2}} .\sqrt 1 }} = \frac{{2\sqrt 3 }}{{\frac{{2\sqrt {87} }}{5}}} = \frac{{5\sqrt {29} }}\).

Suy ra ((A'B'C'), (Oxy)) ≈ 21,8°.

Vậy đáy bể nghiêng so với mặt phẳng nằm ngang một góc khoảng 21,8°.

Mở khóa để xem toàn bộ nội dung trả lời

(?)
Bạn đã đạt đến giới hạn của mình. Bằng cách Đăng ký tài khoản, bạn có thể xem toàn bộ nội dung trả lời
Cải thiện điểm số của bạn bằng cách đăng ký tài khoản Lazi.
Xem toàn bộ các câu trả lời, chat trực tiếp 1:1 với đội ngũ Gia sư Lazi bằng cách Đăng nhập tài khoản ngay bây giờ
Tôi đã có tài khoản? Đăng nhập

Bạn hỏi - Lazi trả lời

Bạn muốn biết điều gì?

GỬI CÂU HỎI
Học tập không giới hạn cùng học sinh cả nước và AI, sôi động, tích cực, trải nghiệm
Bài tập liên quan
Bài tập Toán học Lớp 12 mới nhất

Hôm nay bạn thế nào? Hãy nhấp vào một lựa chọn, nếu may mắn bạn sẽ được tặng 50.000 xu từ Lazi

Vui Buồn Bình thường

Học ngoại ngữ với Flashcard

×
Gia sư Lazi Gia sư
×
Trợ lý ảo Trợ lý ảo